LSAT Logical Reasoning : LSAT Logical Reasoning

Study concepts, example questions & explanations for LSAT Logical Reasoning

varsity tutors app store varsity tutors android store

Example Questions

Example Question #11 : Determining Which Answer Most Strengthens The Argument

Some arguments for expanding U.S. oil production are misguided. Vehicles that do not burn gasoline will be readily available within fifty years, and vehicles that do not burn gasoline do not require oil production. Furthermore, any job creation benefits from increased oil production would be offset by the negative economic effects of producing a product that will not be in demand. Alternatively, jobs can also be created by increased development of oil alternatives.

The argument is strengthened by all of the following, if true, except:

Possible Answers:

Production of alternatives to oil will create as many jobs as the increased production of oil

Current oil production will meet the demand for oil over the next fifty years

When vehicles that do not burn gasoline are readily available, they will be used exclusively

Oil will not be used for any purpose after 50 years

Oil consumption causes substantial environmental harms

Correct answer:

Oil consumption causes substantial environmental harms

Explanation:

The passage only claims that some arguments for increasing oil production are misguided.  It does not indicate that any environmental arguments are misguided. 

Example Question #11 : Strengthen/Support

Residents of a local community started a night watch to reduce crime in their neighborhood. Due in large part to their efforts, no crimes occurred in the neighborhood within the next two months.

Which of the following most strengthens the argument above?

Possible Answers:

The night watch was featured in the local newspaper.

The founder of the night watch was known to everyone in the neighborhood and was a well-respected community leader.

In the two months prior to the night watch, there were ten crimes in the neighborhood, half of which were violent.

Potential criminals in the neighborhood were aware of the night watch.

The night watch membership grew each week during the two month period.

Correct answer:

Potential criminals in the neighborhood were aware of the night watch.

Explanation:

The answer choice that most strengthens the overall argument is the choice stating that potential criminals were aware of the night watch. If potential criminals were not even aware of the night watch, they would probably not be deterred from attempting to commit crimes. If they were aware, they most certainly would not attempt to commit crimes in that neighborhood. In that case, the efforts of the night watch could be said to largely account for the absence of crime. The answer choice discussing the awareness of potential criminals strengthens the overall argument more than the choice discussing the publicity the night watch received in the local newspaper.

Example Question #51 : Lsat Logical Reasoning

The corruption in the city's government began at the very top. While the cash payoffs that were the hallmark of the scheme were only traced to lower-level officials, the payoffs were an open secret at City Hall known to everyone in the building. There is absolutely no possibility that the Mayor and the City Manager did not at least give their tacit approval.

Which of the following, if true, would most strengthen the argument?

Possible Answers:

Such cash payoffs as were found in this scheme are prevalent in many different local governments.

The main official in charge of organizing the cash payoffs swore under oath that the Mayor was unaware of the scheme.

Quite a few recipients of the cash payoffs went against the wishes of the city goverment.

The city has had a long history of low level corruption, beginning well before the current administration.

A memorandum authorizing the cash payoffs was found, with the signatures of both the Mayor and City Manager on it.

Correct answer:

A memorandum authorizing the cash payoffs was found, with the signatures of both the Mayor and City Manager on it.

Explanation:

The main argument is that the corruption scheme is linked directly to the highest levels of the city government in the persons of the Mayor and City Manager. The statement that would most strengthen this argument is one that directly addresses this point. By finding a signed memorandum from the Mayor and City Manager authorizing the payoffs, the argument is reinforced by the evidence.

Example Question #51 : Lsat Logical Reasoning

Video games do nothing to harm children. After being popular for three decades, there is still precious little evidence that playing video games has any negative affect on young brains. In fact, role playing and strategy games should be able to help children develop problem-solving skills better than many other traditional board games.

Which of the following statements would most strengthen the argument of the passage?

Possible Answers:

Scientific research has shown that children who play video games are more successful in school.

Children who play video games more often are more likely to have health problems.

Violent video games introduce children to extreme forms of violence at an early age.

Video games are often the means through which children learn many classic stories.

The most popular video games are those intentionally marketed to children.

Correct answer:

Scientific research has shown that children who play video games are more successful in school.

Explanation:

The passage explicitly states that video games are not harmful to children, and then further speculates that video games could actually be good for children. The statement that would most strengthen this argument is one that further reinforces the good that video games can do, such as scientific evidence showing students who do better in school have played video games.

Example Question #21 : Determining Which Answer Most Strengthens The Argument

A rock band has seen recent declines in their concert attendance. To combat the decline in attendance, the rock band included more new music in their performance because a pop band recently included more new music in their live performances and saw an increase of attendance.

Which of the following most supports the rock band’s reasoning?

Possible Answers:

Rock bands are more popular than pop bands.

People always want to hear a new song.

People who go to see pop bands also go to see rock bands.

Some of the drop in attendance was due to a pop band releasing several new songs.

Rock bands and pop bands have similar patterns of attendance.

Correct answer:

Rock bands and pop bands have similar patterns of attendance.

Explanation:

If rock bands and pop bands have similar patterns of attendance, then it would follow for the rock band to mimic the pop band’s means to increasing attendance. Just because people want to hear a new song does not mean they will necessarily go to a concert to hear it.

Example Question #21 : Determining Which Answer Most Strengthens The Argument

High-caloric food is quite often also the food which people most want to eat. This does not stem from a desire to be unhealthy on the part of humans, but instead reflects that humans have for the most part in history been quite active animals. A desire for calories was a necessity for any humans who would be burning large numbers of calories.

Which of the following statements most strengthens the argument presented in the passage?

Possible Answers:

The highest caloric foods are not necessarily the best sellers in many restaurants.

Human beings like a wide array of foods, both high caloric and low caloric.

High caloric food is among the top sellers in grocery stores across America.

Human activity has been largely consistent across history.

Human beings are less active today than they have ever been in recorded history.

Correct answer:

Human beings are less active today than they have ever been in recorded history.

Explanation:

The argument is best stated as humans prefer high caloric food because humans traditionally burned many calories through activity. This means that anything which strengthens the argument needs to further reinforce the idea that humans were extremely active in their history. The only answer choice that appropriately does this is "Human beings are less active today than they have ever been in recorded history."

Example Question #51 : Lsat Logical Reasoning

Principle: Lectures should not last longer than two hours and will only discuss topics that are relevant to the class as a whole. If none of the topics discussed during the lecture are relevant to a particular student, he or she should not be required to attend the lecture.

Application:  Even though the lecture is only one hour long, Julia should not be required to attend today's lecture.

Which of the following, if true, best justifies the application of the principle above?

Possible Answers:

If Julia attends the lecture, different topics will be relevant to the class as a whole.

The only questions that Julia could ask during the lecture are irrelevant to the class as a whole.

If Julia attends the lecture and asks questions, the lecture will last too long.

Most of the topics discussed in the lecture are not relevant to Julia.

No topics relevant to Julia are relevant to the class as a whole.

Correct answer:

No topics relevant to Julia are relevant to the class as a whole.

Explanation:

The correct answer choice needs to fulfill the following criterion in order to justify Julia not attending class: None of the topics discussed are relevant to Julia. (The application already states that the lecture is less than two hours). None of the answer choices directly state this, but the correct answer choice logically implies this.  If none of the topics that are relevant to Julia are relevant to the class as a whole, then none of the topics relevant to Julia will be discussed (ONLY topics relevant to the class as a whole will be discussed).  And if none of the topics relevant to Julia will be discussed, this justifies Julia not attending the lecture. 

Example Question #22 : Determining Which Answer Most Strengthens The Argument

If the President of Yakistan does not get reelected, then his political party will not maintain power. If the unemployment rate continues to decline, the President of Yakistan will not get reelected. Several experts have predicted that the unemployment rate will continue to decline. Therefore, the President of Yakistan’s political party will not maintain power.

Which of the following, if true, most strengthens the conclusion made in the paragraph above?

Possible Answers:

Yakistan borders Kazubistan which is having economic problems.

The unemployment rate will remain stable.

Expert predictions tend to be reliable.

Several economic experts have political connections with the opposition party to the President of Yakistan.

The President of Yakistan’s political party is likely to select a new platform in the upcoming election.

Correct answer:

Expert predictions tend to be reliable.

Explanation:

We are looking for an answer that strengthens the problem. One of the weaknesses of the stimulus’ argument is that we do not know for sure if we can trust the predictions of the cited experts. The correct answer confirms that the experts are reliable making it much more likely that the unemployment rate will continue to decline. The other answers either weaken the conclusion or have no relevance.

Example Question #51 : Lsat Logical Reasoning

A new government report claims that the current administration has curtailed its domestic surveillance program. However, the report maintains that specific programs within the domestic surveillance infrastructure the government maintains will remain classified. In the past when the government has kept information classified, the visible reforms the government has performed have been negated by secret expansions. Thus, we know that the current administration has failed to curtail its domestic surveillance program.

Each of the following strengthens the argument presented in the paragraph above EXCEPT

Possible Answers:

The government tends to hide its most significant acts of surveillance in classified reports.

The current administration has released several reports with classified information.

The current report is the first the current administration has released.

Classified reports set future precedents for expansion of surveillance.

The only way to curtail the domestic surveillance program is by eliminating secrete expansions in the program.

Correct answer:

The current report is the first the current administration has released.

Explanation:

The correct answer does not strengthen the argument and in fact weakens the argument by showing that the relationship presented that has occurred in the past may not be applicable. The new administration has never released a report with classified information before so we do not know if it will maintain a similar strategy to past administrations. The other answers strengthen the argument presented in the stimulus.

Example Question #21 : Determining Which Answer Most Strengthens The Argument

Intellectual property rights are essential to providing incentives for pharmaceutical companies to innovate and create new medicines. Despite this, the government should be willing to curtail some intellectual property rights. Millions of people are dying of various diseases and the fact that some pharmaceutical companies have exclusive rights to produce certain drugs means many people are unable to purchase those drugs.

Which of the following, if true, most strengthens the argument presented in the stimulus.

Possible Answers:

Some diseases have a long incubation period before they begin to show their symptoms.

Many drugs that are produced by companies other than the ones that initially invented them are defective.

Many pharmaceutical companies would not innovate if they did not have large monetary incentives to do so.

Many of the innovations necessary to help fight against various diseases that are killing millions of people have already occurred and have been patented.

Various countries across the world already violate intellectual property rights.

Correct answer:

Many of the innovations necessary to help fight against various diseases that are killing millions of people have already occurred and have been patented.

Explanation:

The correct answer will help strengthen the argument presented in the stimulus. One way to strengthen this stimulus is by showing that the reduction in innovation produced by curtailing intellectual property rights will not make it harder to help the millions of people across the world dying of various diseases. The correct answer does this by showing that the innovations to help cure the disease have already occurred.

Tired of practice problems?

Try live online LSAT prep today.

1-on-1 Tutoring
Live Online Class
1-on-1 + Class
Learning Tools by Varsity Tutors